Question about a function being continuousA problem on summing real numbers all taken to the same exponentProving a function is continuous on all irrational numbersHow can I prove this function is not continuous for every point other than 0?At what points is this piecewise function continuous?Explain about proofcontinuous but not absolutely continuous functionProb. 17, Chap. 4 in Baby Rudin: A real function can have at most countably many simple discontinuities on a given segmentGiven two real numbers $a$ and $b$ such that $a<b$, what about the convergence of these two sequences?Understanding a functionAlmost everywhere differentiability of a function

RSA: Danger of using p to create q

How much of data wrangling is a data scientist's job?

"You are your self first supporter", a more proper way to say it

Codimension of non-flat locus

Important Resources for Dark Age Civilizations?

Could an aircraft fly or hover using only jets of compressed air?

What defenses are there against being summoned by the Gate spell?

Accidentally leaked the solution to an assignment, what to do now? (I'm the prof)

Approximately how much travel time was saved by the opening of the Suez Canal in 1869?

Was any UN Security Council vote triple-vetoed?

Can a monk's single staff be considered dual wielded, as per the Dual Wielder feat?

Malformed Address '10.10.21.08/24', must be X.X.X.X/NN or

How much RAM could one put in a typical 80386 setup?

Modeling an IP Address

Revoked SSL certificate

Maximum likelihood parameters deviate from posterior distributions

What does "Puller Prush Person" mean?

Why is Minecraft giving an OpenGL error?

Alternative to sending password over mail?

How to determine what difficulty is right for the game?

How does one intimidate enemies without having the capacity for violence?

I'm flying to France today and my passport expires in less than 2 months

Is it possible to run Internet Explorer on OS X El Capitan?

Which country benefited the most from UN Security Council vetoes?



Question about a function being continuous


A problem on summing real numbers all taken to the same exponentProving a function is continuous on all irrational numbersHow can I prove this function is not continuous for every point other than 0?At what points is this piecewise function continuous?Explain about proofcontinuous but not absolutely continuous functionProb. 17, Chap. 4 in Baby Rudin: A real function can have at most countably many simple discontinuities on a given segmentGiven two real numbers $a$ and $b$ such that $a<b$, what about the convergence of these two sequences?Understanding a functionAlmost everywhere differentiability of a function













0












$begingroup$


Let $r_1,r_2,...$ be the set of rational numbers in $[0,1]$ and



$g_n(x)=begincasesfrac1sqrt & xneq r_n; \ 3 & x=r_n.endcases$



This is continuous by definition ?
for all $ϵ>0$, there exists $δ>0$ such that for all $y$, if $|x−y|<δ$, then $|f(x)−f(y)|<ϵ$.










share|cite|improve this question











$endgroup$
















    0












    $begingroup$


    Let $r_1,r_2,...$ be the set of rational numbers in $[0,1]$ and



    $g_n(x)=begincasesfrac1sqrt & xneq r_n; \ 3 & x=r_n.endcases$



    This is continuous by definition ?
    for all $ϵ>0$, there exists $δ>0$ such that for all $y$, if $|x−y|<δ$, then $|f(x)−f(y)|<ϵ$.










    share|cite|improve this question











    $endgroup$














      0












      0








      0





      $begingroup$


      Let $r_1,r_2,...$ be the set of rational numbers in $[0,1]$ and



      $g_n(x)=begincasesfrac1sqrt & xneq r_n; \ 3 & x=r_n.endcases$



      This is continuous by definition ?
      for all $ϵ>0$, there exists $δ>0$ such that for all $y$, if $|x−y|<δ$, then $|f(x)−f(y)|<ϵ$.










      share|cite|improve this question











      $endgroup$




      Let $r_1,r_2,...$ be the set of rational numbers in $[0,1]$ and



      $g_n(x)=begincasesfrac1sqrt & xneq r_n; \ 3 & x=r_n.endcases$



      This is continuous by definition ?
      for all $ϵ>0$, there exists $δ>0$ such that for all $y$, if $|x−y|<δ$, then $|f(x)−f(y)|<ϵ$.







      analysis






      share|cite|improve this question















      share|cite|improve this question













      share|cite|improve this question




      share|cite|improve this question








      edited Mar 29 at 10:49









      J. W. Tanner

      4,4691320




      4,4691320










      asked Mar 29 at 10:29









      kim wenasakim wenasa

      194




      194




















          2 Answers
          2






          active

          oldest

          votes


















          0












          $begingroup$

          $g_n$ is not continuous at $r_n$: if $(x_k)$ is a sequence in $[0,1] setminus r_n$ with $x_k to r_n$ as $k to infty$, then $g_n(x_k) to infty$ as $k to infty.$






          share|cite|improve this answer









          $endgroup$




















            1












            $begingroup$

            It's discontinuous, for $x$ close to $r_n$ values of $frac1sqrt$ get arbitrarily large, so for small $varepsilon$ you won't find such $delta$.






            share|cite|improve this answer









            $endgroup$













              Your Answer





              StackExchange.ifUsing("editor", function ()
              return StackExchange.using("mathjaxEditing", function ()
              StackExchange.MarkdownEditor.creationCallbacks.add(function (editor, postfix)
              StackExchange.mathjaxEditing.prepareWmdForMathJax(editor, postfix, [["$", "$"], ["\\(","\\)"]]);
              );
              );
              , "mathjax-editing");

              StackExchange.ready(function()
              var channelOptions =
              tags: "".split(" "),
              id: "69"
              ;
              initTagRenderer("".split(" "), "".split(" "), channelOptions);

              StackExchange.using("externalEditor", function()
              // Have to fire editor after snippets, if snippets enabled
              if (StackExchange.settings.snippets.snippetsEnabled)
              StackExchange.using("snippets", function()
              createEditor();
              );

              else
              createEditor();

              );

              function createEditor()
              StackExchange.prepareEditor(
              heartbeatType: 'answer',
              autoActivateHeartbeat: false,
              convertImagesToLinks: true,
              noModals: true,
              showLowRepImageUploadWarning: true,
              reputationToPostImages: 10,
              bindNavPrevention: true,
              postfix: "",
              imageUploader:
              brandingHtml: "Powered by u003ca class="icon-imgur-white" href="https://imgur.com/"u003eu003c/au003e",
              contentPolicyHtml: "User contributions licensed under u003ca href="https://creativecommons.org/licenses/by-sa/3.0/"u003ecc by-sa 3.0 with attribution requiredu003c/au003e u003ca href="https://stackoverflow.com/legal/content-policy"u003e(content policy)u003c/au003e",
              allowUrls: true
              ,
              noCode: true, onDemand: true,
              discardSelector: ".discard-answer"
              ,immediatelyShowMarkdownHelp:true
              );



              );













              draft saved

              draft discarded


















              StackExchange.ready(
              function ()
              StackExchange.openid.initPostLogin('.new-post-login', 'https%3a%2f%2fmath.stackexchange.com%2fquestions%2f3166981%2fquestion-about-a-function-being-continuous%23new-answer', 'question_page');

              );

              Post as a guest















              Required, but never shown

























              2 Answers
              2






              active

              oldest

              votes








              2 Answers
              2






              active

              oldest

              votes









              active

              oldest

              votes






              active

              oldest

              votes









              0












              $begingroup$

              $g_n$ is not continuous at $r_n$: if $(x_k)$ is a sequence in $[0,1] setminus r_n$ with $x_k to r_n$ as $k to infty$, then $g_n(x_k) to infty$ as $k to infty.$






              share|cite|improve this answer









              $endgroup$

















                0












                $begingroup$

                $g_n$ is not continuous at $r_n$: if $(x_k)$ is a sequence in $[0,1] setminus r_n$ with $x_k to r_n$ as $k to infty$, then $g_n(x_k) to infty$ as $k to infty.$






                share|cite|improve this answer









                $endgroup$















                  0












                  0








                  0





                  $begingroup$

                  $g_n$ is not continuous at $r_n$: if $(x_k)$ is a sequence in $[0,1] setminus r_n$ with $x_k to r_n$ as $k to infty$, then $g_n(x_k) to infty$ as $k to infty.$






                  share|cite|improve this answer









                  $endgroup$



                  $g_n$ is not continuous at $r_n$: if $(x_k)$ is a sequence in $[0,1] setminus r_n$ with $x_k to r_n$ as $k to infty$, then $g_n(x_k) to infty$ as $k to infty.$







                  share|cite|improve this answer












                  share|cite|improve this answer



                  share|cite|improve this answer










                  answered Mar 29 at 10:39









                  FredFred

                  48.6k11849




                  48.6k11849





















                      1












                      $begingroup$

                      It's discontinuous, for $x$ close to $r_n$ values of $frac1sqrt$ get arbitrarily large, so for small $varepsilon$ you won't find such $delta$.






                      share|cite|improve this answer









                      $endgroup$

















                        1












                        $begingroup$

                        It's discontinuous, for $x$ close to $r_n$ values of $frac1sqrt$ get arbitrarily large, so for small $varepsilon$ you won't find such $delta$.






                        share|cite|improve this answer









                        $endgroup$















                          1












                          1








                          1





                          $begingroup$

                          It's discontinuous, for $x$ close to $r_n$ values of $frac1sqrt$ get arbitrarily large, so for small $varepsilon$ you won't find such $delta$.






                          share|cite|improve this answer









                          $endgroup$



                          It's discontinuous, for $x$ close to $r_n$ values of $frac1sqrt$ get arbitrarily large, so for small $varepsilon$ you won't find such $delta$.







                          share|cite|improve this answer












                          share|cite|improve this answer



                          share|cite|improve this answer










                          answered Mar 29 at 10:39









                          Maja BlumensteinMaja Blumenstein

                          1096




                          1096



























                              draft saved

                              draft discarded
















































                              Thanks for contributing an answer to Mathematics Stack Exchange!


                              • Please be sure to answer the question. Provide details and share your research!

                              But avoid


                              • Asking for help, clarification, or responding to other answers.

                              • Making statements based on opinion; back them up with references or personal experience.

                              Use MathJax to format equations. MathJax reference.


                              To learn more, see our tips on writing great answers.




                              draft saved


                              draft discarded














                              StackExchange.ready(
                              function ()
                              StackExchange.openid.initPostLogin('.new-post-login', 'https%3a%2f%2fmath.stackexchange.com%2fquestions%2f3166981%2fquestion-about-a-function-being-continuous%23new-answer', 'question_page');

                              );

                              Post as a guest















                              Required, but never shown





















































                              Required, but never shown














                              Required, but never shown












                              Required, but never shown







                              Required, but never shown

































                              Required, but never shown














                              Required, but never shown












                              Required, but never shown







                              Required, but never shown







                              Popular posts from this blog

                              Triangular numbers and gcdProving sum of a set is $0 pmod n$ if $n$ is odd, or $fracn2 pmod n$ if $n$ is even?Is greatest common divisor of two numbers really their smallest linear combination?GCD, LCM RelationshipProve a set of nonnegative integers with greatest common divisor 1 and closed under addition has all but finite many nonnegative integers.all pairs of a and b in an equation containing gcdTriangular Numbers Modulo $k$ - Hit All Values?Understanding the Existence and Uniqueness of the GCDGCD and LCM with logical symbolsThe greatest common divisor of two positive integers less than 100 is equal to 3. Their least common multiple is twelve times one of the integers.Suppose that for all integers $x$, $x|a$ and $x|b$ if and only if $x|c$. Then $c = gcd(a,b)$Which is the gcd of 2 numbers which are multiplied and the result is 600000?

                              Ingelân Ynhâld Etymology | Geografy | Skiednis | Polityk en bestjoer | Ekonomy | Demografy | Kultuer | Klimaat | Sjoch ek | Keppelings om utens | Boarnen, noaten en referinsjes Navigaasjemenuwww.gov.ukOffisjele webside fan it regear fan it Feriene KeninkrykOffisjele webside fan it Britske FerkearsburoNederlânsktalige ynformaasje fan it Britske FerkearsburoOffisjele webside fan English Heritage, de organisaasje dy't him ynset foar it behâld fan it Ingelske kultuergoedYnwennertallen fan alle Britske stêden út 'e folkstelling fan 2011Notes en References, op dizze sideEngland

                              Հադիս Բովանդակություն Անվանում և նշանակություն | Դասակարգում | Աղբյուրներ | Նավարկման ցանկ